Making a eigenvector a linear combination of other eigenvectors

Click For Summary
The discussion centers on expressing the eigenvector of the matrix \sigmax with a +1 eigenvalue as a linear combination of the eigenvectors of another matrix M. Participants express confusion regarding how to proceed without knowing the specifics of matrix M. A suggestion is made to denote the eigenvector in question as (v,w) and represent it as a combination of arbitrary eigenvectors (a,b) and (c,d) using constants C1 and C2. Clarification is sought on the identity of matrix M to facilitate the solution. The conversation highlights the necessity of defining M to advance the problem-solving process.
JordanGo
Messages
72
Reaction score
0

Homework Statement


Write the eigenvector of \sigmax with +1 eigenvalue as a linear combination of the eigenvectors of M.


Homework Equations



\sigmax = (0,1),(1,0) (these are the columns)

The Attempt at a Solution



... Don't know what to do. Can someone show me how to do this using arbitrary eigenvectors, say (a,b) and (c,d)?
 
Physics news on Phys.org
Can you find the eigenvector in question? Let's denote it by (v,w). Then you can write
(v,w) = C1(a,b) + C2(c,d) and solve the constant C's.
 
Ok, that makes sense, thanks a lot!
 
JordanGo said:

Homework Statement


Write the eigenvector of \sigmax with +1 eigenvalue as a linear combination of the eigenvectors of M.


Homework Equations



\sigmax = (0,1),(1,0) (these are the columns)
Okay, that's \sigma. What is M? we can't write something "as a linear combination of the eigenvectors of M without knowing what M is!

The Attempt at a Solution



... Don't know what to do. Can someone show me how to do this using arbitrary eigenvectors, say (a,b) and (c,d)?
 
Question: A clock's minute hand has length 4 and its hour hand has length 3. What is the distance between the tips at the moment when it is increasing most rapidly?(Putnam Exam Question) Answer: Making assumption that both the hands moves at constant angular velocities, the answer is ## \sqrt{7} .## But don't you think this assumption is somewhat doubtful and wrong?

Similar threads

  • · Replies 2 ·
Replies
2
Views
2K
  • · Replies 2 ·
Replies
2
Views
2K
  • · Replies 1 ·
Replies
1
Views
5K
  • · Replies 6 ·
Replies
6
Views
2K
Replies
1
Views
1K
  • · Replies 5 ·
Replies
5
Views
2K
Replies
9
Views
2K
  • · Replies 33 ·
2
Replies
33
Views
2K
  • · Replies 12 ·
Replies
12
Views
3K
  • · Replies 5 ·
Replies
5
Views
14K